Board logo

标题: [求助]天山5-7 [打印本页]

作者: leadership    时间: 2005-7-16 08:18     标题: [求助]天山5-7

Q7. For a certain race, 3 teams were allowed to enter 3 members each. A team earned 6-n points whenever one of its memebers finished in nth place, where 1<=n<=5 . There were no ties, disqualifications , or withdrawals. If no team earned more than 6 points, what is the least possible score a team could have earned ?

A. 0

B. 1

C.2

D.3

E.4

answer is : B. I think should be D.

scores are 5,4,3,2,1 . maximum one team can have is 6. thus least score is 5+4+3+2+1-6*2 = 3

高手指正!


作者: assfgrtrtr    时间: 2005-7-16 16:49

正确答案是D.3啊~~GG从哪里看的答案B呢?

晕~~






欢迎光临 国际顶尖MBA申请交流平台--TOPWAY MBA (http://forum.topway.org/) Powered by Discuz! 7.2